Difference between revisions of "2016 AIME II Problems/Problem 15"

m
Line 2: Line 2:
  
 
==Solution==
 
==Solution==
Replace <math>\sum x_ix_j</math> with <math>\frac12\left(\left(\sum x_i\right)^2-\sum x_i^2\right)</math> and the second equation becomes <math>\sum\frac{x_i^2}{1-a_i}=\frac{1}{215}</math>. Conveniently, <math>\sum 1-a_i=215</math> so we get <math>\left(\sum 1-a_i\right)\left(\sum\frac{x_i^2}{1-a_i}\right)=1=\left(\sum x_i\right)^2</math>. This is the equality case of Cauchy so <math>x_i=c(1-a_i)</math> for some constant <math>c</math>. Using <math>\sum x_i=1</math>, we find <math>c=\frac{1}{215}</math> and thus <math>x_2=\frac{3}{860}</math>. Thus, the desired answer is <math>860+3=\boxed{863}</math>.
+
Note that
 +
<cmath>\begin{align*}\sum_{1 \leq i < j \leq 216} x_ix_j &= \frac12\left(\left(\sum_{i=1}^{216} x_i\right)^2-\sum_{i=1}^{216} x_i^2\right)\\&=\frac12\left(1-\sum x_i^2\right).\end{align*}</cmath>
 +
Substituting this into the second equation and collecting <math>x_i^2</math> terms, we find
 +
<cmath>\sum_{i=1}^{216}\frac{x_i^2}{1-a_i}=\frac{1}{215}.</cmath> Conveniently, <math>\sum_{i=1}^{216} 1-a_i=215</math> so we find
 +
<cmath>\left(\sum_{i=1}^{216} 1-a_i\right)\left(\sum_{i=1}^{216}\frac{x_i^2}{1-a_i}\right)=1=\left(\sum_{i=1}^{216} x_i\right)^2.</cmath> This is the equality case of the Cauchy-Schwarz Inequality, so <math>x_i=c(1-a_i)</math> for some constant <math>c</math>. Summing these equations and using the facts that <math>\sum_{i=1}^{216} x_i=1</math> and <math>\sum_{i=1}^{216} 1-a_i=215</math>, we find <math>c=\frac{1}{215}</math> and thus <math>x_2=c(1-a_2)=\frac{1}{215}\cdot \left(1-\frac{1}{4}\right)=\frac{3}{860}</math>. Hence the desired answer is <math>860+3=\boxed{863}</math>.
 +
 
 +
==See Also==
 +
{{AIME box|year=2016|n=II|num-b=14|after=Last Question}}
 +
{{MAA Notice}}

Revision as of 10:27, 17 May 2016

For $1 \leq i \leq 215$ let $a_i = \dfrac{1}{2^{i}}$ and $a_{216} = \dfrac{1}{2^{215}}$. Let $x_1, x_2, ..., x_{216}$ be positive real numbers such that $\sum_{i=1}^{216} x_i=1$ and $\sum_{1 \leq i < j \leq 216} x_ix_j = \dfrac{107}{215} + \sum_{i=1}^{216} \dfrac{a_i x_i^{2}}{2(1-a_i)}$. The maximum possible value of $x_2=\dfrac{m}{n}$, where $m$ and $n$ are relatively prime positive integers. Find $m+n$.

Solution

Note that \begin{align*}\sum_{1 \leq i < j \leq 216} x_ix_j &= \frac12\left(\left(\sum_{i=1}^{216} x_i\right)^2-\sum_{i=1}^{216} x_i^2\right)\\&=\frac12\left(1-\sum x_i^2\right).\end{align*} Substituting this into the second equation and collecting $x_i^2$ terms, we find \[\sum_{i=1}^{216}\frac{x_i^2}{1-a_i}=\frac{1}{215}.\] Conveniently, $\sum_{i=1}^{216} 1-a_i=215$ so we find \[\left(\sum_{i=1}^{216} 1-a_i\right)\left(\sum_{i=1}^{216}\frac{x_i^2}{1-a_i}\right)=1=\left(\sum_{i=1}^{216} x_i\right)^2.\] This is the equality case of the Cauchy-Schwarz Inequality, so $x_i=c(1-a_i)$ for some constant $c$. Summing these equations and using the facts that $\sum_{i=1}^{216} x_i=1$ and $\sum_{i=1}^{216} 1-a_i=215$, we find $c=\frac{1}{215}$ and thus $x_2=c(1-a_2)=\frac{1}{215}\cdot \left(1-\frac{1}{4}\right)=\frac{3}{860}$. Hence the desired answer is $860+3=\boxed{863}$.

See Also

2016 AIME II (ProblemsAnswer KeyResources)
Preceded by
Problem 14
Followed by
Last Question
1 2 3 4 5 6 7 8 9 10 11 12 13 14 15
All AIME Problems and Solutions

The problems on this page are copyrighted by the Mathematical Association of America's American Mathematics Competitions. AMC logo.png